Answer to Question #308559 in Calculus for Amira

Question #308559

Determine whether if lim f(c) = f(c)




x→c





1. f(x) = x+2; c = -1



2. f(x) = x-2; c = 0




3. (at c = -1 )



f(x) = {x ² - 1 if x < -1}



f(x) = { (x - 1) ² - 4 if x ≥ -1}




4. (at c = 1 )



f(x) = {x³ - 1 if x < 1}



f(x) = { x² + 4 if x ≥ 1}




1
Expert's answer
2022-03-15T10:12:10-0400

Solution (a)


"f\\left( x \\right) = x + 2\\\\"


"\\mathop {\\lim }\\limits_{x \\to - 1} f\\left( x \\right) = \\mathop {\\lim }\\limits_{x \\to - 1} \\left( {x + 2} \\right) = - 1 + 2\\\\"


"f\\left( { - 1} \\right) = - 1 + 2\\\\"


"\\mathop {\\lim }\\limits_{x \\to c} f\\left( x \\right) = f\\left( c \\right)"




Solution (b)


"f\\left( x \\right) = x - 2\\\\"


"\\mathop {\\lim }\\limits_{x \\to 0} f\\left( x \\right) = \\mathop {\\lim }\\limits_{x \\to 0} \\left( {x - 2} \\right) = 0 - 2 = - 2\\\\"


"f\\left( 0 \\right) = 0 - 2 = - 2\\\\"


"\\mathop {\\lim }\\limits_{x \\to c} f\\left( x \\right) = f\\left( c \\right)"


Solution (c)


"\\begin{array}{l}\nf\\left( x \\right) = & \\left\\{ \\begin{array}{l}\n{x^2} - 1 & & x < - 1\\\\\n{\\left( {x - 1} \\right)^2} -4& & x \\ge - 1 & \n\\end{array} \\right. & c = - 1\\\\\n\\end{array}\\"



"\\begin{array}{l}\n\n\\mathop {\\lim }\\limits_{x \\to - {1^ - }} f\\left( x \\right) = \\mathop {\\lim }\\limits_{x \\to - {1^ - }} \\left( {{x^2} - 1} \\right) = {\\left( { - 1} \\right)^2} - 1 = 0\\\\\n\\mathop {\\lim }\\limits_{x \\to - {1^ + }} f\\left( x \\right) = \\mathop {\\lim }\\limits_{x \\to - {1^ + }} {\\left( {x - 1} \\right)^2}-4 = {\\left( { - 1 - 1} \\right)^2} -4= 0\n\\end{array}\\"



Since "\\mathop {\\lim }\\limits_{x \\to - {1^ - }} f\\left( x \\right) = \\mathop {\\lim }\\limits_{x \\to - {1^ + }} f\\left( x \\right)\\"


Hence, the limit at "x=c=-1" exists. 



Solution (d)

"\\begin{array}{l}\nf\\left( x \\right) = & \\left\\{ \\begin{array}{l}\n{x^3} - 1 & & x < 1\\\\\n{x^2} + 4 & & x \\ge 1 & \n\\end{array} \\right. & c = 1\\\\\n\\mathop {\\lim }\\limits_{x \\to {1^ - }} f\\left( x \\right) = \\mathop {\\lim }\\limits_{x \\to {1^ - }} \\left( {{x^3} - 1} \\right) = {\\left( 1 \\right)^3} - 1 = 0\\\\\n\\mathop {\\lim }\\limits_{x \\to {1^ + }} f\\left( x \\right) = \\mathop {\\lim }\\limits_{x \\to {1^ + }} \\left( {{x^2} + 4} \\right) = {\\left( {{{\\left( 1 \\right)}^2} + 4} \\right)^2} = 25\n\\end{array}\\"



Since,


"\\mathop {\\lim }\\limits_{x \\to {1^ - }} f\\left( x \\right) \\ne \\mathop {\\lim }\\limits_{x \\to {1^ + }} f\\left( x \\right)\\"


Hence, the limit at "x=c=1" does not exist. 


Need a fast expert's response?

Submit order

and get a quick answer at the best price

for any assignment or question with DETAILED EXPLANATIONS!

Comments

No comments. Be the first!

Leave a comment

LATEST TUTORIALS
New on Blog
APPROVED BY CLIENTS